Location via proxy:   [ UP ]  
[Report a bug]   [Manage cookies]                

PhIMO 2024 FRR S1 Set 3

Download as pdf or txt
Download as pdf or txt
You are on page 1of 24

Secondary 1 Final Round Review Set 3

PART 1: MULTIPLE-CHOICE QUESTIONS

1. Which of the following divides 30222 − 30202 ?


A. 6040 B. 6041 C. 6042 D. 6043 E. 6044

2. The irrational expression 3024 is closest to which of the following?
A. 54.991 B. 54.992 C. 54.993 D. 54.993 E. 54.994

3(934 + 2x)
3. Determine the number of integer solutions that satisfy −2 < < −1.
34
A. 2 B. 3 C. 4 D. 4 E. 6

4. The set of numbers 21, 23, a, 27, 29, b 36, 38 have a median of 31.5 and a range of 18.
Determine the average of these numbers.
5 7 5 7 3
A. 29 B. 29 C. 30 D. 30 E. 31
8 8 8 8 8
5. Which of the following is NOT divisible by (x − 2)?

A. 2x3 − 9x2 + 23x − 26


B. 3x3 + x2 − 10x − 8
C. 2x3 − 11x2 + 7x + 24
D. 7x3 − 11x2 − 21x + 30
E. 2x3 − 13x2 + x + 34

6. The surface area of sphere is four times the product of π and the square of the radius of
the sphere. Determine the proper expression for the diameter of the sphere (d) given its
surface area (A).

r r r r
2A A A A
A. B. C. 2 D. 2 E. 2π A
π π π 2π
7. The mean, median, and mode of a certain list of numbers are equal. Which of the following
must be true?

A. The list must contain an even number of elements.


B. The smallest and largest numbers in the list must be equally spaced from the median.

PhIMO 2024 Final Round


C. The list must contain at least three distinct numbers.
D. The mean must be larger than any individual number in the list.
E. None of these are true.

8. For any positive integer k, which of the following will NOT have an odd number of factors?

A. 4k 2 + 12k + 9
B. 9k 2 − 12k + 4
C. 4k 4 − 12k 2 + 9
D. k 2 + 20k + 36
E. k 2 + 10k + 25

9. Which of the following is the longest?


A. 1. 5 mile B. 2.3 km C. 2300 cm D. 2650 yards E. 7900 ft

10. Which of the following must be true about the product of three irrational numbers?

A. It is always rational.
B. It is always irrational.
C. It can be rational and irrational.
D. It can be real and imaginary number.
E. None of these are true.

PART 2: OPEN-ENDED QUESTIONS


p
11. Simplify 136.1 .

12. Suppose a, b, c, d, e are positive integers such that

1 1640
a+ =
1 709
b+
1
c+
1
d+
e

Determine the value of ab + c + de.

13. A bouncy rubber ball is dropped from a height of 420 centimeters, and each time it strikes
the ground, it bounces back at a height of 57 of the distance from which it fell. Determine
the total vertical distance traveled (in centimeters) by the ball before it comes to rest.

14. Evaluate (12 + 22 + 32 + . . . + 20342 ) − (1 × 3 + 2 × 4 + 3 × 5 + . . . + 2033 × 2035).

PhIMO 2024 Final Round


15. A regular octadecagon is inscribed inside a circle. An inscribed angle is also placed in the
circle, such that its intercepted arc contains three consecutive vertices of the octadecagon.
Determine the measure of the inscribed angle.

16. Simplify (8x3 − 6x2 − 17x + 15) ÷ (4x2 − 9x + 5).


2(x + 3) 3x − 8 3(3x + 5)
17. Determine the value of x if + = .
8 6 4
18. The speed of sound is approximated to be around 340 meters per second. Rounded off to
the nearest hundreds digit, determine its speed in feet per minute.

19. A magic square can be made by arranging numbers 41 to 49 in the 9 places of a 3 × 3


square grid such that all rows, columns, and both diagonals have the same sums. Suppose
three of the arranged numbers, as shown, are related in terms of y. Determine the value
of Z.

5y + 12 4(3y − 7)

y2 + 5

√ √ 2 √ √ 2 −1
 
20. Simplify 11 + 7 − 11 − 7 .

21. Determine the sum of all possible values of x if:


r r
√ √
q q
3x 3x 3x . . . = x+ x+ x + ...

22. Two circular gears are placed side by side and are tangent at point M . The smaller gear
has an inscribed angle of 84◦ , while the larger gear has 9 times the area of the smaller gear.
Suppose the smaller gear is rotated clockwise, making the larger gear rotate accordingly
counterclockwise, until both circles are tangent at P . By how many degrees did the larger
gear rotate in total?

PhIMO 2024 Final Round


23. Ten years ago, a father’s age was equal to the square of his son’s age. In 14 years, the
father will be twice as old as his son. What is the sum of their present ages?

24. Between 5 o’clock and 6 o’clock in the afternoon, the minute hand is before the hour
7
hand. In 14 minutes, the minute hand will be times as far from the hour hand as it
4
was earlier. What time is it?

25. Determine the number of positive integers less than or equal to 3104 that are divisible by
either of 2, 3, 5, 12, 15, 16, 21, or 25.

26. Separate the numbers: 204, 609, 276, 483, 170, and 290 into two groups with equal
products. Determine the average of the numbers in the group which has 290.

27. Given that the inequality 4x + 11 > 7x > 4x + 7 > 16 > x − 3 has solutions on the
a
interval a < x < b. Find the value of .
b
 √ − 2 1

28. Simplify 142 + 30 13 .

7x
29. A parallelogram ABCD has measure of angles ]A = + 23 and ]B = 3(5x − 34).
2
What is the measure of angle ]D?

30. A gambling game is played by guessing one number from 1 to 5. After which, three
random numbers (also from 1 to 5, but not necessarily distinct) are shown.

• If one of the three numbers matches the guessed number, your gambled amount is
returned.
• If two of the three numbers match the guessed number, your gambled amount is
doubled.
• If all three numbers match the guessed number, your gambled amount is tripled.
• If none of the three numbers match the guessed number, you lose your gambled
amount.

If a certain person plays this game multiple times, gambling in the same small amounts
(fraction of his total original money) per game, by what fraction of his original money is
he expected to lose?

PhIMO 2024 Final Round


SOLUTION MANUAL

PART 1: MULTIPLE-CHOICE QUESTIONS

1. Which of the following divides 30222 − 30202 ?


A. 6040 B. 6041 C. 6042 D. 6043 E. 6044

Answer: C
Solution. Recall the formula for the difference of two squares, i.e., a2 − b2 = (a + b)(a − b).
Hence, the given expression can be expressed as,

30222 − 30202 = (3022 + 3020)(3022 − 3020) = 6042(2)

Therefore, 6042 divides 30222 − 30202 .


2. The irrational expression 3024 is closest to which of the following?
A. 54.991 B. 54.992 C. 54.993 D. 54.993 E. 54.994

Answer: A
Solution. Note that 54.9912 ≈ 3024.01. Since 54.991 is the smallest among the choices,
then clearly, the square of all other choices are greater than 3024.01.

3(934 + 2x)
3. Determine the number of integer solutions that satisfy −2 < < −1.
34
A. 2 B. 3 C. 4 D. 4 E. 6

Answer: E
Solution. We have,

3(934 + 2x)
−2 < < −1
34
−68 < 3(934 + 2x) < −34
−68 < 2802 + 6x < −34

PhIMO 2024 Final Round


−68 − 2802 < 6x < −34 − 2802
−2870 < 6x < −2836
2870 2836
− <x<−
6 6
1435 1418
− <x<−
3 3
1 2
−478 < x < −472
3 3

Since we are looking for the integer solution, it follows that x satisfies the integers in the
interval [−478, −473]. Hence, there are −473 − (−478) + 1 = −473 + 478 + 1 = 6.

4. The set of numbers 21, 23, a, 27, 29, b 36, 38 have a median of 31.5 and a range of 18.
Determine the average of these numbers.
5 7 5 7 3
A. 29 B. 29 C. 30 D. 30 E. 31
8 8 8 8 8
Answer: D
Solution. From the known numbers, the difference between the largest and smallest
number is 38 − 21 = 17. Since the range is 18, it follows that one of the unknown number,
say a, is either 20 or 39.
Assume a = 20, then we have the arrangement of known numbers,

20, 21, 23, 27, 29, 36, 38

Note that in this case, it is impossible to insert b in the arrangement such that the median
of the array would be 31.5. Hence, a = 20 is NOT possible.
If a = 39, then we have the arrangement of known numbers,

21, 23, 27, 29, 36, 38 39

Note that if we place b between 29 and 36 (in 5th term), it is possible to get a median of
31.5. That is, if b = 34, then we have the arrangement,

21, 23, 27, 29, 34, 36, 38 39

29 + 34
and the median is given by = 31.5. Hence, this is our final set of numbers, and
2
the average of these numbers is given by,

21 + 23 + 27 + 29 + 34 + 36 + 38 + 39 247 7
= = 30
8 8 8

PhIMO 2024 Final Round


5. Which of the following is NOT divisible by (x − 2)?

A. 2x3 − 9x2 + 23x − 26


B. 3x3 + x2 − 10x − 8
C. 2x3 − 11x2 + 7x + 24
D. 7x3 − 11x2 − 21x + 30
E. 2x3 − 13x2 + x + 34

Answer: C
Solution. By Factor Theorem, (x − 2) is a factor of any polynomial P (x), if P (2) = 0.
Hence, we want to look at the polynomial below which gives as P (2) 6= 0. We have the
following:

a. 2(2)3 − 9(2)2 + 23(2) − 26 = 16 − 36 + 46 − 26 = 0


b. 3(2)3 + (2)2 − 10(2) − 8 = 24 + 4 − 20 − 8 = 0
c. 2(2)3 − 11(2)2 + 7(2) + 24 = 16 − 44 + 14 + 24 = 10 6= 0

Therefore, (x − 2) is NOT a factor of 2x3 − 11x2 + 7x + 24.

6. The surface area of sphere is four times the product of π and the square of the radius of
the sphere. Determine the proper expression for the diameter of the sphere (d) given its
surface area (A).

r r r r
2A A A A
A. B. C. 2 D. 2 E. 2π A
π π π 2π
Answer: B
Solution. Given A is the surface area of the sphere, we use the fact that diameter is twice
the radius, i.e., d = 2r. We then have,

A
A = 4πr2 =⇒ r2 =
r4π
A
r=
r4π
1 A
r=
2 πr
A
=⇒ d = 2r =
π

PhIMO 2024 Final Round


7. The mean, median, and mode of a certain list of numbers are equal. Which of the following
must be true?

A. The list must contain an even number of elements.


B. The smallest and largest numbers in the list must be equally spaced from the median.
C. The list must contain at least three distinct numbers.
D. The mean must be larger than any individual number in the list.
E. None of these are true.

Answer: E
Solution. Consider the following reasoning:

(a) The list {1, 3, 3, 3, 5} has equal mean, median, and mode which is 3, however, has
no even number element. Hence, choice A is not necessary.
(b) The list {1, 1, 3, 3, 3, 4, 6} has 3 as mean, median, and mode. But 6 − 3 6= 3 − 1.
Hence, choice B is not necessary.
(c) The list {3, 3, 3} has equal mean, mode, and median but has only one number in the
list. Hence, choice C is not necessary.
(d) Following example in (c) the mean is not larger than any of the other element in the
list. Hence, choice D is not necessary.

8. For any positive integer k, which of the following will NOT have an odd number of factors?

A. 4k 2 + 12k + 9
B. 9k 2 − 12k + 4
C. 4k 4 − 12k 2 + 9
D. k 2 + 20k + 36
E. k 2 + 10k + 25

Answer: D
Solution. For a number to have an odd number of factors, the number must be a per-
fect square. Otherwise, the number will have an even number of factors. Consider the
following:

a 4k 2 + 12k + 9 = (2k + 3)2


b. 9k 2 − 12k + 4 = (3k − 2)2
c. 4k 4 − 12k 2 + 9 = (2k 2 − 3)2
d. k 2 + 20k + 36 = (k + 18)(k + 2)

PhIMO 2024 Final Round


e. k 2 + 10k + 25 = (k + 5)2

Clearly, the only non-perfect square among the choices is k 2 + 20k + 36. Hence, it will
not have an odd number of factors for any integer k.

9. Which of the following is the longest?


A. 1. 5 mile B. 2.3 km C. 2300 cm D. 2650 yards E. 7900 ft

Answer: D
Solution. We convert all measurement in terms of centimeters:
160934 cm
(a) 1.5 mile × = 241401 cm
1 mile
100000 cm
(b) 2.3 km × = 230000 cm
1 km
91.44 cm
(d) 2650 yards × = 242316 cm
1 yard
30.48 cm
(e) 7900 ft × = 240792 cm
1 ft

Clearly, the longest is 2650 yards.

10. Which of the following must be true about the product of three irrational numbers?

A. It is always rational.
B. It is always irrational.
C. It can be rational and irrational.
D. It can be real and imaginary number.
E. None of these are true.

Answer: C
Solution. Consider the following reasoning:
√ √ √
(a) Take the three irrational numbers 2, 3, and 5. The product of these three
√ √ √ √
numbers is 2 × 3 × 5 = 30, which is irrational. Hence, choice A is false.
√ √ √ √ √
(b) Consider the irrational number 2. Then, 2 × 2 × 2 = 8 = 2, which is
3 3 3 3 3

rational. Hence, choice B is false.


(c) Combining (a) and (b) means the product of three irrational numbers can be rational
and can be irrational. Hence, choice C is true.

PhIMO 2024 Final Round


(d) Note that irrational numbers are real numbers. By closure property of real numbers,
we know the product of any irrational numbers is always a real number, i.e., it cannot
be an imaginary number. Hence, choice D is false.

PART 2: OPEN-ENDED QUESTIONS


p
11. Simplify 136.1 .
35 2
Answer: or 11
3 3
1
Solution. Note that 136.1 = 136 . Then,
9
r
p 1
136.1 = 136
9
r
1225
=
9

1225
= √
9
35 2
= or 11
3 3

12. Suppose a, b, c, d, e are positive integers such that

1 1640
a+ =
1 709
b+
1
c+
1
d+
e

Determine the value of ab + c + de.


Answer: 53
1640
Solution. Consider . Then we have,
709
1640 222
=2+
709 709
1
=2+
709
222
1
=2+
43
3+
222

PhIMO 2024 Final Round


1
=2+
1
3+
222
43
1
=2+
1
3+
7
5+
43
1
=2+
1
3+
1
5+
43
7
1
=2+
1
3+
1
5+
1
6+
7

Hence, a = 2, b = 3, c = 5, d = 6, and e = 7. It follows,

ab + c + de = 2(3) + 5 + 6(7) = 6 + 5 + 42 = 53

13. A bouncy rubber ball is dropped from a height of 420 centimeters, and each time it strikes
the ground, it bounces back at a height of 57 of the distance from which it fell. Deter-
mine the total vertical distance traveled (in centimeters) by the ball before it comes to rest.
Answer: 2520
Solution. The vertical distance traveled by the bouncy ball is given by,

5 5 5 5 5 5
420+ (420)+ (420)+ (300)+ (300)+. . .+0 = 420+ (420+420)+ (300+300)+. . .+0
7 7 7 7 7 7

Consider the expression,


 
5 5 5 5 5
(420 + 420) + (300 + 300) + . . . + 0 = 600 + (600) + (600) + . . . + 0
7 7 7 7 7

5
Let r be the common ratio, then, |r| = < 1 which means the series converges into a
7
sum. If a is the first term, then the geometric sum is given by,

a 600 600 7(600)


S∞ = = = = = 7(300) = 2100
1−r 5 2 2
1−
7 7

Therefore, the bouncy rubber ball traveled a total distance of 420 + 2100 = 2520.

PhIMO 2024 Final Round


14. Evaluate (12 + 22 + 32 + . . . + 20342 ) − (1 × 3 + 2 × 4 + 3 × 5 + . . . + 2033 × 2035).
Answer: 2034
Solution. Recalling the formula for the sum of squares of first n natural numbers, we have

n(n + 1)(2n + 1) 2034(2035)(4069)


12 + 22 + 32 + . . . + 20342 = = (1)
6 6

On the other hand, the expression 1 × 3 + 2 × 4 + 3 × 5 + . . . + 2033 × 2035 is just the


sum of the first k terms of the form k(k + 2) = k 2 + 2k.
k(k + 1)
Note the formula for the sum of first k natural numbers is .
2
Thus, we can express,
 
k(k + 1)(2k + 1) k(k + 1)
1 × 3 + 2 × 4 + 3 × 5 + . . . + 2033 × 2035 = +2
6 2
2033(2034)(4067)
= + 2033(2034) (2)
6

Subtracting equation (2) from equation (1), we have


 
2034(2035)(4069) 2033(2034)(4067)
− + 2033(2034)
6 6
 
2034(2035)(4069) 2033(2034)(4067) + 6(2033)(2034)
= −
6 6
2034(2035)(4069) − [2033(2034)(4067) + 6(2033)(2034)]
=
6
2034(2035)(4069) − 2034[2033(4067) + 6(2033)]
=
6
2034(2035)(4069) − 2034[2033(4067 + 6)]
=
6
2034(2035)(4069) − 2034[2033(4073)]
=
6
2034(8280415 − 8280409)
=
6
2034(6)
=
6
=2034

Alternative Solution. Note from the given above, n = 2034 and k = 2033. Hence,
k = n − 1. Then, we can simply have,
  
n(n + 1)(2n + 1) k(k + 1)(2k + 1) k(k + 1)
− +2
6 6 2
  
n(n + 1)(2n + 1) (n − 1)(n)(2(n − 1) + 1) (n − 1)n
= − +2
6 6 2

PhIMO 2024 Final Round


(n2 + n)(2n + 1)
 2  2 
(n − n)(2n − 1) n −n
= − +6
6 6 6
2n + n + 2n + n 2n − n − 2n + n 6n2 − 6n
3 2 2 3 2 2
= − −
6 6 6
2n3 + n2 + 2n2 + n − 2n3 + n2 + 2n2 − n − 6n2 + 6n
=
6
2n + n + 2n + n − 2n + n2 + 2n2 − n − 6n2 + 6n
3 2 2 3
=
6
6n
=
6
= n = 2034

15. A regular octadecagon is inscribed inside a circle. An inscribed angle is also placed in the
circle, such that its intercepted arc contains three consecutive vertices of the octadecagon.
Determine the measure of the inscribed angle.
Answer: 20
Solution. The octadecagon (18-sided polygon) is inscribed in a circle means that the
vertices of the said polygon lie on the circle. Since the octadecagon is regular, then the
central angle subtended by an arc between any two adjacent vertices is given by,

360
= 20
18

This can be illustrated by the figure below:

Hence, the inscribed angle subtended by an arc that contains three consecutive vertices
is
1
(2)(20) = 20
2

PhIMO 2024 Final Round


16. Simplify (8x3 − 6x2 − 17x + 15) ÷ (4x2 − 9x + 5).
Answer: 2x + 3
Solution. By long division,

2x + 3
4x2 − 9x + 5 8x3 − 6x2 − 17x + 15
−(8x3 − 18x2 + 10x)
12x2 − 27x + 15
−(12x2 − 27x + 15)
0

2(x + 3) 3x − 8 3(3x + 5)
17. Determine the value of x if + = .
8 6 4
26 8
Answer: − or −2
9 9
Solution. We have,

2(x + 3) 3x − 8 3(3x + 5)
+ =
8 6 4
3[2(x + 3)] + 4(3x − 8) = 6[3(3x + 5)] (multiply 24)
3(2x + 6) + (12x − 32) = 6(9x + 15)
6x + 18 + 12x − 32 = 54x + 90
18x − 14 = 54x + 90
54x − 18x = −14 − 90
36x = −104
104 26 8
x=− = − = −2
36 9 9

18. The speed of sound is approximated to be around 340 meters per second. Rounded off to
the nearest hundreds digit, determine its speed in feet per minute.
Answer: 66900
Solution. Using 1 m = 3.281 ft, we have,

340 m
 3.281 ft 60 sec
 340 × 3.281 × 60 ft 66932 ft
× × = = ≈ 66900
1 sec
 1m
  1 min 1 min 1 min

PhIMO 2024 Final Round


19. A magic square can be made by arranging numbers 41 to 49 in the 9 places of a 3 × 3
square grid such that all rows, columns, and both diagonals have the same sums. Suppose
three of the arranged numbers, as shown, are related in terms of y. Determine the value
of Z.

5y + 12 4(3y − 7)

y2 + 5

Answer: 47
Solution. In a 3 × 3 magic square, the midpoint of all numbers from 41 to 49 must be at
the center square. Hence, the number in the center must 45.
Also, the sum of every rows, columns and diagonals are all equal to

41 + 42 + 43 + 44 + 45 + 46 + 47 + 48 + 49 405
= = 135
3 3

Let the square r1 c2 = n (first row, second column), then it follows that,

y 2 + 5 + 45 + n = 5y + 12 + n + 4(3y − 7)
y 2 + 50 = 5y + 12 + 12y − 28
y 2 − 5y − 12y + 50 − 12 + 28 = 0
y 2 − 17y + 66 = 0
(y − 11)(y − 6) = 0

Note that if y = 11, then the square r1 c1 = 5y + 12 = 5(11) + 12 = 67 which is not


possible under our condition (i.e., numbers are until 45 only). Hence, we take y = 6.
Then, we have the following:

r1 c1 = 5y + 12 = 5(6) + 12 = 30 + 12 = 42
r1 c3 = 4(3y − 7) = 4[3(6) − 7] = 4(18 − 7) = 4(11) = 44
r3 c1 = 135 − (45 + r1 c3 ) = 135 − (45 + 44) = 135 − 89 = 46

PhIMO 2024 Final Round


Therefore,

Z = 135 − (r1 c1 + r3 c1 ) = 135 − (42 + 46) = 135 − 88 = 47

√ √ 2 √ √ 2 −1
 
20. Simplify 11 + 7 − 11 − 7 .

77
Answer:
308
Solution. Recall the formulas: (a + b)2 = a2 + 2ab + b2 and (a − b)2 = a2 − 2ab + b2 . Then,

1 1
√ √ 2 √ √ 2 =  √ √   √ √ 
11 + 7 − 11 − 7 11 + 2 11 · 7 + 7 − 11 − 2 11 · 7 + 7

1
= √   √ 
11 + 2 11 · 7 + 7 − 11 − 2 11 · 7 + 7
1
= √ √
11 + 2 77 + 7 − 11 + 2 77 − 7
1
= √ √
2 77 + 2 77

1 77
= √ ×√
4 77 77
√ √
77 77
= =
4(77) 308

21. Determine the sum of all possible values of x if:


r r
√ √
q q
3x 3x 3x . . . = x+ x+ x + ...

4
Answer:
9
r r
√ √
q q
Solution. Let y = 3x 3x 3x . . . and z = x + x + x + . . . . Then, we have,

p
y= 3xy =⇒ y 2 = 3xy
=⇒ y = 3x (1)

On the other hand,



z= x+z =⇒ z2 = x + z =⇒ z2 − z = x

PhIMO 2024 Final Round


1 1
=⇒ z2 − z + = x +
4 4
 2
1 4x + 1
=⇒ z− =
2 4
r
1 4x + 1
=⇒ z− =±
2 4

1 4x + 1
=⇒ z= ±
2 √ 2
1 ± 4x + 1
=⇒ z= (2)
2

From our assumption, we can clearly equate equations (1) and (2). We have,

1±4x + 1
3x =
2

6x − 1 = ± 4x + 1
 √ 2
(6x − 1)2 = ± 4x + 1

36x2 − 12x + 1 = 4x + 1
36x2 − 16x = 0
4x(9x − 4) = 0

Hence our possible values for x are:

4
4x = 0 =⇒ x = 0 and 9x − 4 = 0 =⇒ x =
9
4 4
Therefore, the sum of possible values is, 0 + = .
9 9

22. Two circular gears are placed side by side and are tangent at point M . The smaller gear
has an inscribed angle of 84◦ , while the larger gear has 9 times the area of the smaller gear.
Suppose the smaller gear is rotated clockwise, making the larger gear rotate accordingly
counterclockwise, until both circles are tangent at P . By how many degrees did the larger
gear rotate in total?

PhIMO 2024 Final Round


Answer: 56
Solution. Let AS and RS be the area and radius of the smaller square, respectively. Then,
r
2 AS
AS = πRS =⇒ RS =
π

It follows that, if RL is the radius of the larger circle (which area is 9 times the smaller
circle), then r r
2 9AS AS
9AS = πRL =⇒ RL = =3
π π
Hence, the radius of the larger circle is 3 times the smaller one, i.e., RL = 3RS .
If both circles rotate at opposite direction such that they will be tangent from M to P ,
means that the arc length, traveled by both gears are equal. Recall that the formula for
determining arc length of a circle is,

π
ArcLength = r × θ × (where θ is central angle in degrees subtended by the arc)
180
_ _
Note that the central angle subtended by M P (or the angular measure of M P ) is twice
the inscribed angle it subtends. Let this central angle be θS , then

θS = 2 × 84◦ = 168◦

Now, let θL be the central angle in the larger circle subtended by the arc which length is
_
equal to the length of M P in the smaller circle, then we have

π π
RS × θS × = RL × θL ×
180 180
π π
RS × 168 × = 3RS × θL ×
180 180
168 = 3θL
168
θL = = 56
3

23. Ten years ago, a father’s age was equal to the square of his son’s age. In 14 years, the
father will be twice as old as his son. What is the sum of their present ages?
Answer: 62
Solution. Let x and y be the age of the father and son, respectively. Then we can have
these equations,
(
x − 10 = (y − 10)2 =⇒ x − 10 = y 2 − 20y + 100 =⇒ y 2 − 20y − x + 110 = 0
x + 14 = 2(y + 14) =⇒ x + 14 = 2y + 28 =⇒ x = 2y + 14

PhIMO 2024 Final Round


Substitute x from the 2nd equation to the 1st equation,

y 2 − 20y − x + 110 = 0
y 2 − 20y − (2y + 14) + 110 = 0
y 2 − 20y − 2y − 14 + 110 = 0
y 2 − 22y + 96 = 0
(y − 16)(y − 6) = 0
y = 16 ; y = 6

But we must reject y = 6 since the son’s age will be −4 ten years ago. Hence, we take
y = 16. It follows that,

x = 2y + 14 = 2(16) + 14 = 32 + 14 = 46

Therefore, x + y = 46 + 16 = 62.

24. Between 5 o’clock and 6 o’clock in the afternoon, the minute hand is before the hour
7
hand. In 14 minutes, the minute hand will be times as far from the hour hand as it
4
was earlier. What time is it?
2
Answer: 5 : 22
11
Solution. Note that the angle created between minute and hour hand, given that the
minute hand is before the hour hand is given by

11 11 11
θ = 30H − M = 30(5) − M = 150 − M (1)
2 2 2
7
In 14 minutes, the angle between minute and hour hand would be × θ. Also, the minute
4
hand should now be after the hour hand. Hence, we have

7θ 11 11
= (M + 14) − 30(5) = (M + 14) − 150 (2)
4 2 2

Substituting (1) to (2), we therefore have,


 
7 11 11
150 − M = (M + 14) − 150
4 2 2
1050 77 11 154
− M= M+ − 150
4 8 2 2
525 154 11 77
− + 150 = M+ M
2 2 2 8

PhIMO 2024 Final Round


525 − 154 + 300 44 + 77
= M
2 8
121 671
M=
8 2
671 8
M= ×
2 121
671(4) 11(61)(4) 244 2
= = = or 22
121 11(11) 11 11

2
Therefore, it is 5 : 22 in the afternoon
11

25. Determine the number of positive integers less than or equal to 3104 that are divisible by
either of 2, 3, 5, 12, 15, 16, 21, or 25.
Answer: 2276
Solution. Note that all numbers divisible by 12 or 16 are divisible by 2, numbers that are
divisible by 15 or 21 are divisible by 3, and all numbers divisible by 25 are also divisible
by 5. Hence, we can just count how many numbers less than or equal to 3104 are divisible
by 2, 3, and 5. We have,
     
3104 3104 3104
+ + = 1552 + 1034 + 620 = 3206
2 3 5

The number of pairwise intersection to be discounted is,


     
3104 3104 3104
+ + = 517 + 310 + 206 = 1033
2×3 2×5 3×5

Then, we want to count back the LCM(2, 3, 5) = 30,


 
3104
= 103
30

Therefore, we have,
3206 − 1033 + 103 = 2276

26. Separate the numbers: 204, 609, 276, 483, 170, and 290 into two groups with equal prod-
ucts. Determine the average of the numbers in the group which has 290.
977 2
Answer: or 325
3 3

PhIMO 2024 Final Round


Solution. We express all numbers in terms of prime factors,

204 = 22 × 3 × 17
609 = 3 × 7 × 29
276 = 22 × 3 × 23
483 = 3 × 7 × 23
170 = 2 × 5 × 17
290 = 2 × 5 × 29

In order for the two groups to have equal product the pairs with common prime factors
can’t be on the same group. By inspection, we determine the grouping (204 , 483 , 290)
and (170 , 276 , 609 , ), as illustrated below:

(204 × 483 × 290) − (170 × 276 × 609)


=[(22 × 3 × 17)(3 × 7 × 23)(2 × 5 × 29)] − [(2 × 5 × 17)(22 × 3 × 23)(3 × 7 × 29)]
=(23 × 32 × 5 × 7 × 17 × 23 × 29) − (23 × 32 × 5 × 7 × 17 × 23 × 29) = 0

Therefore the average of the numbers in the group having 290 is

204 + 483 + 290 977 2


= = 325
3 3 3

27. Given that the inequality 4x + 11 > 7x > 4x + 7 > 16 > x − 3 has solutions on the
a
interval a < x < b. Find the value of .
b
7
Answer:
11
Solution. Break the inequality into four parts:

11
4x + 11 > 7x =⇒ 3x < 11 =⇒ x < (1)
3
7
7x > 4x + 7 =⇒ 3x > 7 =⇒ x > (2)
3
9
4x + 7 > 16 =⇒ 4x > 9 =⇒ x > (3)
4
16 > x − 3 =⇒ x < 16 + 3 =⇒ x < 19 (4)

11
Combining inequalities (1) and (4), their intersection is x < . While the intersection
3

PhIMO 2024 Final Round


7
of (2) and (3) is x > . Thus, the solution to the given inequality is
3
7 11 7 11
<x< =⇒ a= and b =
3 3 3 3

Therefore, we have
7
a 7
= 3 =
b 11 11
3

 √ − 12
28. Simplify 142 + 30 13 .

3 13 − 5
Answer:
92
Solution. We write,

1 1
√ = √
q q
142 + 30 13 25 + 30 13 + 117
1
=q p
25 + 2 152 (13) + 117
1
=q √
25 + 2 2925 + 117
1
=q p
25 + 2 25(117) + 117
1
=q √
(5 + 117)2
1
=q √
(5 + 3 13)2

1 3 13 − 5
= √ × √
5 + 3 13 3 13 − 5

3 13 − 5
=
9(13) − 25

3 13 − 5
=
92

PhIMO 2024 Final Round


7x
29. A parallelogram ABCD has measure of angles ]A = + 23 and ]B = 3(5x − 34).
2
What is the measure of angle ]D?
Answer: 108
Solution. Note that adjacent angles of a parallelogram are supplementary. Then,

7x
]A + ]B = + 23 + 3(5x − 34) = 180
2
7x
+ 23 + 15x − 102 = 180
2
7x + 30x
= 180 − 23 + 102
2
37x = 2(259)
37x = 518
x = 14

Now, we know that opposite angles of parallelogram are congruent. Hence, ∠B ∼


= ∠D,
and it follows that,

]D = ]B = 3(5x − 34) = 15x − 102 = 15(14) − 102 = 210 − 102 = 108

30. A gambling game is played by guessing one number from 1 to 5. After which, three
random numbers (also from 1 to 5, but not necessarily distinct) are shown.

• If one of the three numbers matches the guessed number, your gambled amount is
returned.
• If two of the three numbers match the guessed number, your gambled amount is
doubled.
• If all three numbers match the guessed number, your gambled amount is tripled.
• If none of the three numbers match the guessed number, you lose your gambled
amount.

If a certain person plays this game multiple times, gambling in the same small amounts
(fraction of his total original money) per game, by what fraction of his original money is
he expected to lose?
2
Answer:
5
Solution. We first determine the probabilities under each case:

PhIMO 2024 Final Round


Case 1: The probability that none of the three numbers match the guessed number is:
 3
4 64
=
5 125

Case 2: The probability that one of the three numbers match the guessed number is:
   2
3 1 4 16 48
× × =3× 2
=
1 5 5 5×5 125

Case 3: The probability that two of the three numbers match the guessed number is:
   2
3 1 4 4 12
× =3× 2 =
2 5 5 5 ×5 125

Case 4: The probability that all of the three numbers match the guessed number is:
 3
1 1
=
5 125

Let x be the gambled amount. Then, in Case 1, player’s gain is −x; in Case 2, player’s
gain is 0; in Case 3, player’s gain is x; and in Case 4, player’s gain is 2x. Hence,

64 48 12 1 64x 12x 2x
(−x) + (0) + (x) + (2x) = − +0+ +
125 125 125 125 125 125 125
−64x + 12x + 2x
=
125
50
=− x
125
2
=− x
5
2
This means that the expected loss per game is of the gambled amount x.
5

PhIMO 2024 Final Round

You might also like